PLZ HELP WILL GIVE BRANLIEST IF ANSWER IS CORRECT AND 30 POINTS

PLZ HELP WILL GIVE BRANLIEST IF ANSWER IS CORRECT AND 30 POINTS

Answers

Answer 1

Answer: sooo ok i see here that it might be 5 inches if i'm wrong correct me  

Step-by-step explanation:


Related Questions

Ryan is helping his fellow students build a float for a parade. They need to create a giant spoon for their float. They base their design on a spoon that is 7.25 inches long. If each inch of the model spoon represents 2 feet of the giant spoon, how long will the giant spoon be, in feet?​

Answers

7.25*2=14.5

If each inch equals 2 feet you just need to multiply by the factor of (2)

Can some please help me w this and explain it to me

Answers

BCD= 120 degrees. explanation: the 4 angles of a parallelogram must equal 360 degrees. so you add all 4 angles & get 6x. you then take that 6 & divide it from 360 (360/6) and get 60. therefore x=60. so then you just multiple BCD (2x) by 60 and you get 120 degrees
2x+ x =180 degree( being co interior angle)
3x =180
x=180/3
x=60 degree
Here ,
Angle BCD=2x
=2*60
=120 degree

Anabelle picked 65 apples at a local orchard. 40% of the apples were green apples. How many green apples did Annabelle pick?​

Answers

Answer:

she picked 26 green apples

Step-by-step explanation:

Which one should I choose

Answers

Answer:

it depends what are your choices

Mark has 338 inches of Christmas lights. How many feet of Christmas lights does he have?​

Answers

Answer:

28.1667 ft

Step-by-step explanation:

there are 12 inches in a foot so jsut divide 338 by 12

Answer:

the answer is in the picture below :3

Order of operations with and without variables

Answers

Answer: 4

Step-by-step explanation:

when you replace the variables with their numbers, you would get:

5+3/5-3 =

8/2 =

4

hope this helps lol

Answer:

[tex]\frac{a+b}{a-b}=4[/tex]

Step-by-step explanation:

Given the expression

[tex]\frac{a+b}{a-b}[/tex]

Given the values

a=5b=3

substituting the values in the expression

[tex]\frac{a+b}{a-b}=\frac{5+3}{5-3}[/tex]

[tex]\mathrm{Add\:the\:numbers:}\:5+3=8[/tex]

       [tex]=\frac{8}{5-3}[/tex]

[tex]\mathrm{Subtract\:the\:numbers:}\:5-3=2[/tex]

        [tex]=\frac{8}{2}[/tex]

[tex]\mathrm{Divide\:the\:numbers:}\:\frac{8}{2}=4[/tex]

           [tex]=4[/tex]

Thus,

[tex]\frac{a+b}{a-b}=4[/tex]                

help pls. the question is in the picture at the top.

Answers

Answer:

solve for 6x+8

Answer:

x = 3

Step-by-step explanation:

Since ∠ P and ∠ S are congruent then the triangle is isosceles and

PT = TS , substitute values

7x + 4 = 5x + 10 ( subtract 5x from both sides )

2x + 4 = 10 ( subtract 4 from both sides )

2x = 6 ( divide both sides by 2 )

x = 3

-1/5 × (-5/6 )

help,​

Answers

Answer: 1/6 or 0.16

Step-by-step explanation:

How many 1/8's are in 1¼?

Answers

Answer:

there are 2 1/8

Step-by-step explanation:

plz brainllest

f
(
x
)
=
x
2

2
x

8
on
[

1
,
3
]

Answers

Answer:

Step-by-step explanation:

3x + 5y = 7

Since that is our original form, let's convert it so that we can find the slope:

5y = -3x + 7

y = -3/5 x + y

 

 

From here, we use the point-slope equation and then convert that into slope-intercept form:

y - y1 = m(x - x1)

y - 6 = 5/3(x - 0)

y - 6 = 5/3x

what is the combined cost of of the sweatshirt and hat shown
sweatshirt: $25.15
hat: $19.74

Answers

Answer:

$44.89

Step-by-step explanation:

Add 25.15 + 19.74 = 44.89

Answer:

$44.89

Step-by-step explanation:

since the question says combined we know to add the two numbers

25.15+19.74=44.89

so the correct answer would be $44.89

Janet can make a
of a necklace in 20 minutes. At this rate, how many necklaces, to the nearest tenth of a necklace, can Janet make in 1 hour?

Answers

Answer:

she can make 4 necklaces . theres 60 minutes in an hour if it takes 20 minutes she can make 4 . 4x20=60

Step-by-step explanation:

Answer:

3/5

Step-by-step explanation:

A paraboloid container is being filled with fluid at the rate of 4.5 cubic feet per minute. At what rate is the level of fluid changing when the depth
is 2.5 feet? The ratio of the radius to the height of the container is 3:7.
"V = 1/2 pir^2h

Answers

Answer:

2.62 ft./min

Step-by-step explanation:

Just took the test.

Heh sorry I need help again...
David wants to save $186 for a trip to an amusement park. He sets aside $12 of his allowance at the end of each week. How many weeks will it take him to save enough money for the whole trip?

Answers

Answer:

It will take him 15 and a half weeks.

Step-by-step explanation:

186 ÷ 12

Answer:

12*7=?

Step-by-step explanation:You have too count 12 ,7 times t

In a dance contest in which 53,000 votes were cast, the winner received 3/5 of votes. How many votes did the winner not receive? (Incorrect solution: 3/5 x 53,000 = 3/5 x 53,000/1 = 3x53,000/ 5 x 1 = 31,800) ( explain error please and add correct work/solution and if you could share a strategy you used? Thanks!!)

Answers

Answer:

21,200

Step-by-step explanation:

Votes casted = 53000

Votes received = 3/5

Votes did not receive = 1- 3/5 = (5-3)/5 = 2/5

Total votes not received by winner

= 53000 *2/5

= 10,600 * 2

= 21,200

while working at the school store, john sold a jacket for $40.00 and notebooks for $1.50 each.If he collected $92.50, how many notebooks did he sell?

Answers

Answer:

35 Notebooks

Step-by-step explanation:

$1.50x35=52.5

$52.5+$40=$92.5

or $92.50 - 40=$52.5

35x1.5=52.5

Answer:

35 notebooks

Step-by-step explanation:

let notebook as be 'x'

here, $40.00+x($1.50)=$92.50

x($1.50)=$92.50-$40.00

x($1.50)=$52.50

x=$52.50/$1.50

x=35

what is the equation in slope-intercept form for the line that passes through the points (-1,3) and (-2,-3)

A. Y=6x-9

B.y=6x+9

C. Y=-6x-9

D. Y= -6x+9

Answers

the answer is B. 6x+9
my work is attached :)

WILL GIVE BRAINLIESTHow could you use an equation in your future employment?

Answers

When the interviewer asks what the simplified form of the equation 4x^5/78-x=y you have to answer, just in case yk.. but other than that.. hmm maybe to find the ratio of paper to pencil when u work :D

In which of the tables below x and y are inversely proportional? Find the constant of variation. PLZ PLZ PLZ PLZ PLZ PLZ HELP WILL GIVE 5 STARS AND THANKS AND MARK BRAINLYIST
(a)
x 3 4 5 5.5
y 8 6 4.8 4

(b)
x 0.1 0.5 75 100
y 300 60 0.4 0.3

HELP plz :( :((((((((

Answers

The answer is B

Step-by-step explanation:

the table B represent an inverse variation, the constant of variation k is 30

(-9, -5) and (-1, -9)
find the midpoint

Answers

Answer:

(-5,-7)

Step-by-step explanation:

whats is the slope. Help asp.

Answers

the slope would be -1

Tasha wants to find the quotient of 1026 divided by 27. She creates

Answers

Answer:

tasha wants to find the quotient of 1026 divided 27. she creates a table to show the products of 27 multiplied by different multiples of 10

Step-by-step explanation:

I need to find the area of these if you do it I will give you a 5-star rating and share you with other people brainiest included

Answers

Answer:

get up a calculator and multiply the numbers and it will give you an answer

Step-by-step explanation:

hope this helps

Which of the following statements says that a number is between -3 and 3?
Ix) = 3
Ix<3
O x > 3

Answers

Answer:

The inequality that represents this condition would be:

-3 < x < 3

Step-by-step explanation:

Let x be the number between -3 and 3

Since x is between  -3 and 3, so we do not have to include the integer -3 and 3 in the interval.

In inequality:

The symbol '>' is used to represent 'greater than'.The symbol '<' is used to represent 'less than'.

Since the number 'x' is between -3 and 3. It means the number must be greater than -3 and less than 3.

Thus, the inequality that represents this condition would be:

-3 < x < 3

The graph of the line number involving the solution -3 < x < 3 is also attached below.

Therefore, we conclude that:

[tex]-3<x<3\quad :\quad \begin{bmatrix}\mathrm{Solution:}\:&\:-3<x<3\:\\ \:\mathrm{Interval\:Notation:}&\:\left(-3,\:3\right)\end{bmatrix}[/tex]

Please answer please

Answers

Answer:3 units up four units to the left

Step-by-step explanation:

what the quotient of
[tex] \frac{3}{4} \div \frac{1}{8} [/tex]

Answers

The correct answer is 6!

On which triangle can the law of cosines be used to find the length of an unknown side? Law of cosines: a2 = b2 + c2 – 2bccos(A)

Answers

Explanation:

You would use this law under two conditions:

One angle and two edges defined, while trying to solve for the third edgeALL three edges defined

Hence, you have your answer.

* Just make sure to use the inverse function towards the end, or elce you will throw your answer off!

_______________________________________________

Now, you would use the Law of Sines under three conditions:

Two angles and one edge defined, while trying to solve for the second edgeOne angle and two edges defined, while trying to solve for the second angleALL three angles defined [of which does not occur very often, but it all refers back to the first bullet]

* I HIGHLY suggest you keep note of all of this significant information. You will need it going into the future.

I am delighted to assist you at any time.

Solve the system of equations 2 + 2y = -20 and x + 3y = -27 by combining the
equations.

Answers

Answer:

no estoy segura como se debe responder

1 (x + 2y = -20) ----> x*2y= -20

-1 (x + 3y = -27) ----> x-3y= 27

          (0) x + (-1) y = 7

A big box retailer has one order for 9 plants and 6 bushes with a cost of $390. There is a second order for 7 plants and 8 bushes with a
cost of $400.
Which system of equations can be used to determine the price of a plant, p, and the price of a bush, b?

9p +6b = 390
7p + 8b = 400

9p + 8b = 390
7p + 6b = 400

7p + 8b = 390
9p + 6b = 400

7p +95 = 390
6p+8b = 400

Answers

Answer:

Here you go( I dont feel like taking a screen shot- sorry)

Step-by-step explanation:

Very helpful for k12 students, and hope this helps! sorry I am late.

9p+6b=390

7p+8b=400

(I took the test on this and got it right!)

use either the multiples method or the prime factors method to find the least common multiple of 24 and 16.

Answers

The prime factorization of 16: 16 = 2 ⋅ 2 ⋅ 2 ⋅ 2 = 24.

The prime factorization of 24: 24 = 2 ⋅ 2 ⋅ 2 ⋅ 3 = 23 ⋅ 3.

Both 16 and 24 have 23 ( 8 ) in common, so we can remove it from one of the numbers, leaving 24 and 3 . There are no more numbers in common, so the lowest common multiple is 24 ⋅ 3 = 48.

Other Questions
PLZ HELP, WILL MARK BRAINLEST Select the correct answer.Post Exam : Question 24From which land did Arabia get materials such as silk and porcelain?O SpainO ChinaO IndiaO Persia O North AfricaGIVING BRAINLIEST! What creates a hole in the graph of a rational function? The equation Y-3 = -2(x+ 5) is writen in point-slope form. What is the y-intercept of the ine?O 13O-7O28 A librarian sorts 500 books at a constant rate. After 4 days, the librarian still has 250 books to sort. How much time does it take to sort the books from start to finish? find the value of x.- 5- 5 radical 2- 50- radical 5 The span was so long, the height so great, and the enterprise likely to be so costly, that few thought of it as something begun in earnest. . . . But Science said, "It is possible," and Courage said, "It shall be!" . . . . And so this Bridge is a wonder of science.Opening Ceremonies of theNew York and Brooklyn Bridge,Seth LowWhich phrase from the excerpt is a fact that supports the opinion that the bridge is a wonder of science?the span was so long, the height so greatScience said, It is possiblesomething begun in earnest Which ethnic group was forced to abandon their nomadic lifestyle after World War I?Kurds, Persians, or Arabs what is parrel to the equation y=-1/3-x-1 as water warms on the stove, what is happening to the molecules in the liquid? At the county fair, the Baxter family bought 6 hot dogs and 4 juice drinks for $16.70. The Farley family bought 3 hot dogs and 4 juice drinks for $10.85.Which system of equations correctly represents this situation where x equals the price of a hot dog, and y equals the price of a juice drink?A) 6x+4y=10.853x+4y=16.70B)6x+4y=16.703x+4y=10.85C) 4x+6y=16.704x+3y=10.85D) 6x+3y=10.854x+4y=16.70 If each angle in equilateral triangle measures 5X +10 then find x HELP I WILL GIVE BRAINLIEST IF CORRECTmultiple choiceWhich of the following expressions is equalvalent to -3.5(2-3n)-2.5nA.-7-8nB.-7+8nC.-7-13nD-7-n (10.5 - 2.5)E.-7+n(10.5 - 2.5 Select the correct answer.The hand reaching forward with the palm up is the sign for what call?A. Ball outB. Illegal hit / carryC. PointD. Double hit I NEED HELP ASAP PLZ! Which organisms undergo photosynthesis? What are the possible gametes for AaBB? La ecuacin ordinaria de la circunferencia cuyo centro es (-1,5) y su radio es -2/3 es: Energy from______ and other nutrients is transferred to _______ through a series of chemical reactions? Can someone pls explain and do this solution:(